0
$\begingroup$

Given $S \in \mathbb{N}^{q\times n}$ for $q < n $ natural numbers, is there a pseudo-polynomial algorithm which can decide whether exists $x \in \{0,1\}^n$ such that $$ S \cdot x = \begin{bmatrix}t_1\\ ... \\ t_q \end{bmatrix} \in \mathbb{N}^q $$ ? Is it possible to modify for instance the dynamic programming for the subset sum problem?

$\endgroup$

1 Answer 1

1
$\begingroup$

We can reduce 3-color to this problem with all elements being $0$ or $1$, so there is no pseudo-polynomial algorithm unless P = NP.

For each pair of (vertex, color) and (edge, color) we will have a column in $S$ (and position in $x$) such that colorings will correspond to $x$ having ones on positions corresponding to $(v_i, c_j)$ if $v_i$ is colored in $c_j$, and to $(e_i, c_j)$ if none of vertices incident to $e_i$ is colored in $c_j$.

Rows in $S$ (and elements in $t$) will correspond to vertices, edges and (color, edge) pair.

Column corresponding to $(v_i, c_j)$ has one in row corresponding to $v_i$, and also in all rows corresponding to $(e_k, c_j)$ for edges $e_k$ incident to $v_i$.

Column corresponding to $(e_i, c_j)$ has one in row corresponding to $e_i$ and one in row corresponding to $(e_i, c_j)$.

$t$ is just vector of all ones.

To get $1$ in $t$ in position corresponding to $v_i$, we need to have $1$ in exactly one positions of $x$ corresponding to $v_i$. So $x$ defines some coloring.

And the coloring is correct, because if we have two adjacent vertices colored in the same color, then position in $t$ corresponding to (edge between these vertices, color) will be $2$.

$\endgroup$
2
  • $\begingroup$ Thank you for answering! Will assuming $S \in \{0,1\}^{q \times n}$ help anyhow? In the sense that if a PTAS exists and the elements in the matrix $S$ are smaller than $\log(n)$, then can we take $\epsilon = \frac{1}{\log(n)}$ to obtain the exact solution and a complexity in $\mathcal{O}(n^{\log(n)})$ or so $\endgroup$ Commented May 17, 2024 at 18:29
  • $\begingroup$ My reduction gives $S \in \{0, 1\}^{q \times n}$. I know next to nothing about approximations though. $\endgroup$ Commented May 18, 2024 at 1:46

You must log in to answer this question.

Start asking to get answers

Find the answer to your question by asking.

Ask question

Explore related questions

See similar questions with these tags.